Teilbarkeit von 3^n - 2^n durch 2^ceil(log2(3)*n) - 3^n

Neue Frage »

Jack22 Auf diesen Beitrag antworten »
Teilbarkeit von 3^n - 2^n durch 2^ceil(log2(3)*n) - 3^n
Meine Frage:
Sei die Menge der natürlichen Zahlen (für diese Aufgabe ohne Null).
Sei die Menge der ungeraden Zahlen und die Menge der geraden Zahlen (für diese Aufgabe sind mit (un-)geraden Zahlen stets (un-)gerade natürliche Zahlen größer Null gemeint).
Variable n: ,
Variable m: .
Finde ein n, sodass ODER
zeige, dass für alle n gilt: .

Meine Ideen:


Da sowohl der Nenner als auch der Zähler ungerade Zahlen sind, ist das Resultat, wenn es eine natürliche Zahl ist, eine ungerade Zahl.
Selbstverständlich ist jede ungerade Zahl eine natürliche Zahl.

Eine ungerade Zahl plus eins ist gerade Zahl und eine gerade Zahl minus eins ist eine ungerade Zahl. Da die Null ausgeschlossen ist, muss der Sonderfall nicht beachtet werden.



Da der Nenner ungerade ist, kann dessen Primfaktorzerlegung keinen Faktor 2 enthalten, d.h. damit der gesamte Bruch eine gerade Zahl ist, muss sein.
Ist , dann ist offensichtlich eine gerade Zahl.


Ich habe dann den Graph von online angeschaut und es sieht so aus, als ob der Bruch für am größten (nämlich 1) ist und die Werte generell Richtung Null gehen (https://www.desmos.com/calculator/nqm1xelpln) .
Allerdings weiß ich nicht, wie ich es beweisen kann.


Weitere Ansätze, die vielleicht Ideen liefern:
I)





II)
Da eine irrationale Zahl ist, ist auch eine irrationale Zahl.
Somit ist .
frac(x) ist dabei der Nachkommaanteil und kann laut englischem Wikipedia (https://en.wikipedia.org/wiki/Floor_and_ceiling_functions#Continuity_and_series_expansions) so berechnet werden:

.
HAL 9000 Auf diesen Beitrag antworten »

Das gehört anscheinend zu den Problemen, die sich sehr leicht formulieren aber ungeheuer schwer beweisen lassen:

Warum sollte es nicht doch große ganzzahlige sowie zugehöriges geben, so dass bzw. gar gilt?

Für ist das ja der Fall, aber größere zu finden gelingt anscheinend nicht. Aber ein Beweis ist das nicht, und ein Zugang zur Abschätzung dieser Differenz nach unten scheint ein Fall für ganz ausgefuchste Zahlentheoretiker. verwirrt
Jack22 Auf diesen Beitrag antworten »

Hallo Hal,

ich bin heute zufällig auf diesen Beitrag gestoßen:
mathoverflow questions/116840/distance-between-powers-of-2-and-powers-of-3 .

Es gibt also eine untere Abschätzung (Corollary 1.8) für :


,

wobei c eine von a und b abhängige Konstante ist.
Stimmt ein Kommentar im Beitrag, so gilt: .



Da und einen Zähler mit polynomialen Wachstum, aber einen Nenner mit exponentiellen Wachstum hat, ist auch der zweite Term ab einem gewissen n kleiner als 1 .

Grobe Abschätzung für das n:

, was für der Fall ist.

Alle können also per Program überprüft werden, um die Frage zu beantworten.

Trotzdem Danke!
Neue Frage »
Antworten »



Verwandte Themen

Die Beliebtesten »
Die Größten »
Die Neuesten »